Tải bản đầy đủ (.pdf) (71 trang)

Một số bài toán hình học tổ hợp

Bạn đang xem bản rút gọn của tài liệu. Xem và tải ngay bản đầy đủ của tài liệu tại đây (1.02 MB, 71 trang )

ĐẠI HỌC QUỐC GIA HÀ NỘI
TRƢỜNG ĐẠI HỌC KHOA HỌC TỰ NHIÊN
_______________******______________

VŨ MINH HẢI

MỘT SỐ BÀI TỐN
HÌNH HỌC TỔ HỢP

LUẬN VĂN THẠC SĨ KHOA HỌC

Hà nội - 2015

1


ĐẠI HỌC QUỐC GIA HÀ NỘI
TRƢỜNG ĐẠI HỌC KHOA HỌC TỰ NHIÊN
_______________******______________

VŨ MINH HẢI

MỘT SỐ BÀI TỐN
HÌNH HỌC TỔ HỢP
Chun ngành: Phương pháp toán sơ cấp.
Mã số: 60460113.
LUẬN VĂN THẠC SĨ KHOA HỌC
NGƢỜI HƢỚNG DẪN KHOA HỌC
PGS.TS. LÊ ANH VINH

Hà nội – 2015



2


MỤC LỤC
CHƢƠNG 1. BÀI TỐN PHỦ HÌNH .....................................................................5
1.1. Một số lý thuyết cơ sở ................................................................................................ 5
1.2. Một số bài toán phủ hình .......................................................................................... 6

CHƢƠNG 2. BÀI TỐN ĐỒ THỊ, TƠ MÀU ......................................................19
2.1. Lý thuyết cơ bản về bài tốn tơ màu ....................................................................... 19
2.2. Phương pháp tơ màu giải bài tốn hình học.......................................................... 20
2.2.1. Một số bài tốn tơ màu đồ thị ........................................................................ 20
2.2.2. Một số bài tốn tơ màu ơ vng ..................................................................... 37
2.2.3. Một số bài tốn dùng phƣơng pháp tơ màu ơ vng và tính chất bất biến41

CHƢƠNG 3. NGUYÊN LÝ CỰC HẠN ................................................................47
3.1. Nguyên lý cực hạn ................................................................................................... 47
3.2. Ứng dụng nguyên lý cực hạn .................................................................................. 47
3.2.1. Một số bài tốn đánh giá góc.......................................................................... 47
3.2.2. Một số bài toán đánh giá khoảng cách, độ dài ............................................. 54
3.2.3. Một số bài tốn đánh giá diện tích, thể tích .................................................. 63

1


LỜI NĨI ĐẦU
Các bài tốn hình học tổ hợp là các bài toán hay và được nhiều người quan
tâm. Trong những đề thi học sinh giỏi quốc gia và quốc tế cũng thường xun
xuất hiện những bài tốn về hình học tổ hợp. Đó là lý do luận văn này trình

bày một số bài tốn về hình học tổ hợp.
Luận văn “Một số bài tốn hình học tổ hợp” được chia làm 3 chương:
Chƣơng 1. Trình bày một số lý thuyết về bài tốn phủ hình và cách giải
những bài tốn dạng đó.
Chƣơng 2. Trình bày về các bài tốn đồ thị, tơ màu và một số bài tốn
thuộc dạng này được sử dụng trong các kì thi học sinh giỏi trong nước và
quốc tế.
Chƣơng 3. Trình bày về nguyên lý cực hạn và các bài tốn hình học tổ
hợp sử dụng nguyên lí cực hạn.
Mục đích của luận văn là trình bày ngắn họn dễ hiểu lý thuyết về các bài
tốn : phủ hình, đồ thị, tơ màu, bài tốn sử dụng ngun lý cực hạn và trình
bày chi tiết cách giải các bài tốn đó.
Mặc dù có nhiều cố gắng trong việc nghiên cứu và thực hiện luận văn này
nhưng khơng thể tránh khỏi có sai sót, kính mong được sự góp ý q báu của
các thầy, cơ và các bạn. Tôi xin chân thành cảm ơn.

4


CHƢƠNG 1. BÀI TỐN PHỦ HÌNH
Bài tốn phủ hình là một dạng bài tốn có nhiều trong thực tế. Ví dụ như là
việc lát vỉa hè, quảng trường, sàn nhà, ... bằng những viên gạch đa giác giống
nhau.
Câu hỏi được đặt ra ở đây là “Những viên gạch đa giác lồi giống nhau như
thế nào thì có thể lát kín được mặt phẳng?”. Mặt phẳng được lấp đầy bởi
những đa giác giống nhau sao cho hai đa giác tuỳ ý khơng có điểm chung,
nhưng có thể có chung cạnh chung đỉnh.
Từ câu hỏi trên có một số các dạng tốn được sinh ra, đó là “Phủ hình
bằng mạng lưới ơ vuông”, “Phủ đa giác lồi bằng những đa giác vị tự (hoặc
đồng dạng) với chính nó”, ...

Dưới đây luận văn sẽ trình bày một số định lí, hệ quả và những bài tốn
cho dạng bài tốn phủ hình như thế.
1.1. Một số lý thuyết cơ sở
Một hệ thống vô hạn ô vuông tạo nên mặt phẳng được gọi là mạng lưới
đỉnh ơ vng. Các ơ vng đó được gọi là ô vuông cơ sở. Các đỉnh ô vuông là
các điểm ngun (điểm có cả tung độ và hồnh độ là các số nguyên) của một
hệ trục toạ độ song song với các cạnh hình vng cơ sở và có đơn vị gốc là độ
dài cạnh hình vng cơ sở. Một đa giác có đỉnh là các đỉnh lưới của mạng ơ
vng được gọi là đa giác ngun.
Ta có một tính chất cơ bản của mạng lưới ô vuông là định lí sau
Định lí 1. Đa giác đều duy nhất có đỉnh tại các điểm lưới ơ vng là hình
vng.
Mạng lưới ô vuông có một ứng dụng khá thực tế đó là sử dụng để tính diện
tích các hình phẳng. Ta có các định nghĩa sau
Đa giác nguyên : Đa giác có đỉnh là các điểm có toạ độ nguyên.

5


Tam giác đơn: Tam giác có các đỉnh có toạ độ nguyên mà không chứa
đỉnh nguyên nào bên trong hoặc trên cạnh của nó.
Định lí 2. Diện tích của tam giác đơn trên mạng lưới ô vuông đơn vị đúng
bằng

1
.
2

Định lí 3. (Định lí Picard)
Các đỉnh của một đa giác P có cạnh khơng tự cắt (khơng nhất thiết phải

lồi) nằm ở các điểm nguyên. Bên trong nó có n điểm ngun, cịn trên biên m
điểm ngun. Khi đó diện tích của nó bằng
SP  n 

m
1
2

1.2. Một số bài tốn phủ hình
Sau đây luận văn trình bày một số bài tốn phủ hình. Những bài tốn này
được tham khảo ở các tài liệu [1], [2] và [4] mục tài liệu tham khảo.
Bài tốn 1. Trên một tờ giấy có một vết mực diện tích nhỏ hơn 1. Chứng
minh rằng ta có thể kẻ carơ tờ giấy với các hình vng đơn vị (cạnh 1) sao cho
khơng có đỉnh của mạng lưới ô vuông nào rơi vào vết mực cả.
Giải.
Giả sử ta phủ tờ giấy bằng một mạng lưới ô vng đơn vị bất kì.
Sau đó nếu đem cắt các ô vuông đơn vị rời ra và xếp chồng lên nhau. Giả
sử phần ơ vng bị thấm mực có thể thấm thẳng qua tất cả các ơ vng đó.
Khi đó vì diện tích của vết mực nhỏ hơn 1. Nên trong ơ vng có ít nhất 1
điểm là khơng bị thấm mực. Ta đánh dấu điểm đó.
Ta đem trải các ô vuông đó ra như cũ. Các điểm được đánh dấu như trên sẽ
tạo thành một lưới ô vuông phủ tờ giấy mà khơng có điểm nào trong chúng
nằm trong vết mực.
Vậy bài toán được giải. 

6


Bài tốn 2. Cho tam giác nhọn ABC có diện tích bằng 1. Chứng minh rằng
tồn tại một tam giác vng có diện tích khơng vượt q 3 phủ kín ABC .

A
Giải. Gọi BC là cạnh lớn nhất của
tam giác nhon ABC có diện tích bằng

R

1. Kẻ trung tuyến AM. Đặt MA = R.
D

Vẽ đường tròn (M;R) cắt BC ở D, E.

B

H M

a

C

E

Ta có DAE  90 .
0

Các điểm B, C đối xứng nhau qua M, chúng cùng nằm trong đường trịn.
Ta chứng minh rằng tam giác vng ADE là tam giác phải tìm.
Rõ ràng ADE phủ ABC , cần chứng minh S ADE  3 . Kẻ đường cao AH.
Đặt MB = MC = a. Ta có
S ADE 


2S
1
2 1
DE. AH  R. AH ; AH  ABC 

2
BC
2a a

nên S ADE 

R
.
a

Ta chứng minh

R
 3.
a

Thật vậy, trong hai góc AMB, AMC tồn tại một góc lớn hơn hoặc bằng 900 ,
chẳng hạn AMC  900 , do đó
AM 2  MC 2  AC 2 .

Suy ra
R2  a 2  AC 2  BC 2  4a 2
 R 2  3a 2 

R

 3.
a

Vậy tam giác ADE vng có diện tích khơng vượt q 3 phủ kín ABC . 

7


Bài tốn 3. Một khu vực dân cư có hình tứ giác lồi. Tại trung điểm mỗi
cạnh tứ giá, người ta đặt một trung tâm phát và nhận sóng. Vùng phát sóng và
nhận sóng lớn nhất là hình trịn có đường kính là cạnh đó của tứ giác. Có thể
khẳng định rằng toàn bộ khu vực dân cư ấy đều được phủ sóng hay khơng?
Giải.

A

Giả sử có điểm M nằm trong khu

B

dân cư có hình tứ giác lồi ABCD mà
khơng bị phủ bởi hình trịn nào như

M

hình vẽ.
D

C


Lúc đó do M nằm ngồi cả 4 đường trịn có đường kính AB, BC, CD, DA
nên AMB  900 , BMC  900 , CMD  900 , DMA  900.
Suy ra tổng bốn góc trên nhỏ hơn 3600 , vơ lí.
Vậy khơng tồn tại điểm M như thế. Hay có thể khẳng định là khu dân cư
ấy đều được phủ sóng. 
Bài toán 4. Cho 100 điểm trên mặt phẳng, hai điểm nào cũng có khoảng
cách khơng q 1, ba điểm nào cũng là đỉnh của một tam giác tù. Chứng minh
rằng tồn tại một hình trịn có bán kính

1
phủ 100 điểm đã cho.
2

Giải.
Gọi A, B là hai điểm có khoảng cách
lớn nhất trong 100 điểm đã cho, ta có
AB  1. Vẽ đường trịn có đường kính
AB, hình trịn này có bán kính khơng
q

1
. Ta chứng minh rằng hình trịn
2

đó chứa mọi điểm đã cho.

8


Thật vậy, vẽ hai đường thẳng vng góc với AB tại A và tại B tạo thành

một dải.
Nếu

tồn

tại

một

điểm

C

đã

cho

nằm

ngồi

dải

thì

hoặc BC > AB hoặc AC > AB, trái với cách chọn hai điểm A, B.
Nếu tồn tại một điểm C đã cho nằm trên dải và nằm ngồi hình trịn thì 
ABC khơng có góc tù, trái với đề bài. 
Bài tốn 5. Cho bốn điểm trên mặt phẳng, hai điểm nào cũng có khoảng
cách lớn hơn 1. Chứng minh rằng khơng thể phủ tất cả bốn điểm ấy bởi một

hình trịn có đường kính khơng q 2 .
Giải. Ta sẽ chứng minh rằng trong bốn điểm đã cho, tồn tại hai điểm có
khoảng cách lớn hơn 2 . Xét ba trường hợp :
a) Bốn điểm A, B, C, D là đỉnh của
một tứ giác lồi. Tồn tại một góc lớn
hơn hoặc bằng 900 , chẳng hạn A . Ta
sẽ chứng minh BC  2 .
Thật vậy, do A  900 nên
BC 2  AB2  AC 2  1  1  2 .

Vậy BC  2 .
b) Ba điểm (chẳng hạn A, B, C) là đỉnh của một tam giác, điểm thứ tư D
nằm trong hoặc trên biên.
 Nếu D nằm trên biên của tam giác,
chẳng hạn D nằm giữa A và C thì AD > 1,
DC > 1 nên AC > 2 >

2.

9


 Nếu D nằm trong  ABC thì trong
ba góc ADB, BDC, CDA , tồn tại một góc
lớn hơn hoặc bằng 1200 , giả sử
CDA  1200 . Dễ thấy AC  2 .

c) Bốn điểm A, B, C, D thẳng hàng.
Bài toán hiển nhiên đúng. 
Bài toán 6. Cho một đa giác đơn (khơng nhất thiết lồi) có chu vi 12. Chứng

minh rằng có thể phủ kín đa giác bởi một hình trịn có bán kính 3.
Giải. Gọi A, B là hai điểm thuộc biên của đa giác chia chu vi của đa giác
thành hai phần bằng nhau, mỗi phần có độ dài 6. Ta có AB < 6. Gọi O là trung
điểm của AB. Vẽ hình trịn (O ; 3), ta sẽ chứng minh rằng hình trịn này phủ
kín đa giác.
Chứng minh bằng phản chứng. Giả sử tồn
tại điểm C thuộc biên của đa giác mà nằm
ngoài đường trịn thì OC > 3. Điểm C chia
biên của đường gấp khúc từ A đến B thành
hai phần có độ dài m, n. Thế thì AC  m,
CB  n nên
AC + CB  m + n = 6

(1)

Mặt khác, lấy C' đối xứng với C qua O, ta có :
AC + CB = AC + AC'  CC' > 6

(2)

Dễ thấy (1) và (2) mâu thuẫn. Vậy mọi điểm của đa giác đều thuộc hình
trịn (O ; 3). 

10


Bài toán 7. Cho một đa giác lồi. Xếp được nhiều nhất 10 đồng xu hình trịn
có đường kính 1, đơi một khơng giao nhau và có tâm nằm trong đa giác.
Chứng minh rằng 10 hình trịn có bán kính 1 đồng tâm với 10 đồng xu sẽ phủ
kín đa giác.

Giải. Giả sử 10 hình trịn lớn (bán
kính 1) khơng phủ kín đa giác (h. 200)
thế thì tồn tại một điểm A nằm ngồi
các hình trịn lớn và nằm trong đa
giác.
Như vậy cịn đặt được thêm một đồng xu có đường kính 1 có tâm A mà
khơng cắt các đồng xu trước, tức là số đồng xu có đường kính 1 khơng cắt
nhau và có tâm nằm trong đa giác lớn hơn 10, trái với giải thiết.
Vậy bài toán được chứng minh.
Bài toán 8. Cho một đa giác lồi. Xếp được nhiều nhất 20 hình trịn có bán
kính 1, đơi một khơng giao nhau và có tâm nằm trong đa giác, xếp được ít
nhất n hình trịn có bán kính 2 phủ kín đa giác. Chứng minh rằng n  20.
Giải. Vẽ 20 hình trịn có bán kính 2 đồng tâm với 20 hình trịn có bán kính 1
đã xếp. 20 hình trịn lớn này phủ kín đa giác (chứng minh tương tự bài tốn 8).
Điều đó chứng tỏ n  20. 
Bài tốn 9. Bên trong một hình chữ nhật có diện tích 35, đặt năm tam giác,
mỗi tam giác có diện tích 9. Chứng minh rằng tồn tại hai tam giác có diện tích
phần chung khơng nhỏ hơn 1.
Giải. Gọi các tam giác là D1 , D2 ,..., D5 . Giả sử diện tích phần chung của hai
tam giác nào cũng nhỏ hơn 1.
D1 có diện tích bằng 9.

11


Phần diện tích của D2 khơng bị phủ bởi D1 lớn hơn
9  1 = 8.
Phần diện tích của D3 không bị phủ bởi D1 , D2 lớn hơn
9  1  1 = 7.
Phần diện tích của D4 khơng bị phủ bởi D1 , D2 , D3 lớn hơn

9  1  1  1 = 6.
Phần diện tích của D5 không bị phủ bởi D1 , D2 , D3 , D4 lớn hơn
9  1  1  1  1 = 5.
Diện tích của hợp D1 , D2 , D3 , D4 , D5 lớn hơn
9 + 8 + 7 + 6 + 5 = 35.
Điều này vơ lí. Vậy bài tốn được chứng minh. 
Bài tốn 10. Trong mặt phẳng cho một số hữu hạn điểm không cùng nằm
trên một đường thẳng. Chứng minh rằng tồn tại một đường tròn đi qua ba
điểm trong chúng và nó phủ tất cả các điểm cịn lại.
Giải.
Trước tiên ta có chú ý sau :
Cho đoạn thẳng AB và hai điểm M và N không thuộc đoạn thẳng trên. Ta
xét trường hợp khi nào thì bán kính r của đường trịn ngoại tiếp tam giác ABN
lớn hơn bán kính k của đường tròn ngoại tiếp tam giác ABM.
a) Cho ANB và AMB là những góc nhọn. Khi đó dễ thấy rằng r > k chỉ khi
ANB < AMB . Điều này vẫn đúng khi cả hai góc khơng là tù.

12


b) Nếu một trong các góc là tù, giả sử là ANB , cịn góc kia khơng tù, thì ta
sẽ có r > k khi và chỉ khi ANB > 1800 - AMB .
c) Nếu cả hai góc đều là tù, thì r > k khi ANB > AMB .
Quay trở lại bài toán.
Ta dựng bao lồi P của tập điểm đã cho. Vì các điểm khơng cùng nằm trên
một đường thẳng nên bao lồi của chúng không phải là một đoạn thẳng.
Ta xét tất cả các tam giác có những đỉnh giữa các đỉnh của P và chọn tam
giác có bán kính đường trịn ngoại tiếp lớn nhất. Ta cho đó là tam giác ABC.
Giả sử tồn tại đỉnh N của P nằm
ngồi đường trịn ngoại tiếp tam giác


α
M

N

ABM. Khi đó N nằm ngồi ABM.
Suy ra tồn tại ít nhất 1 đỉnh của tam
giác ABM, giả sử là B sao cho N và B
nằm ở hai nửa mặt phẳng khác nhau
đối với cạnh AM của nó. Cần chú ý là

A
α

B α

vì tính lồi của P nên khơng một đỉnh
nào có thể nằm ở các vùng  .
Hơn nữa một trong những góc BAM và BMA là góc tù.
Ta cho đó là BAM .
Khi đó MNB  BAM , điều này có nghĩa là bán kính đường trịn ngoại tiếp
tam giác MNB lớn hơn bán kính đường trịn ngoại tiếp BAM, điều này trái
với cách chọn tam giác ABM. Vậy khơng tồn tại đỉnh N nằm ngồi đường
trịn ngoại tiếp tam giác ABM. 

13


Bài toán 11. Chứng minh rằng mỗi đa giác lồi có diện tích bằng 1 có thể

phủ bằng một hình bình hành có diện tích bằng 2.
Giải. Ta kí hiệu C là đỉnh xa nhất đối với một cạnh của đa giác M. Đường
thẳng AC chia M thành hai phần (hoặc chỉ một phần) M1 và M2. Cho D1 là
đỉnh của M1 và D2 là đỉnh của M2 sao cho D1 và D2 là những đỉnh xa nhất đối
với AC.

C

S

R

D2

D1

P

B

A

Q

Qua C, D1, D2 lần lượt dựng các đường thẳng song song với AB, AC, AC.
Ta có được hình bình hành PQRS.
1
2

1

2

Ta có S ACD  S ACSP , S ACD  S AQRC .
1

2

1
2

Do đó S AD CD  S PQRS .
1

2

Dễ thấy đa giác M có diện tích lớn hơn diện tích đa giác AD1CD2, mặt khác
nếu diện tích đa giác AD1CD2 là 1 thì diện tích đa giác PQRS là 2.
Vậy kết luận một đa giác có diện tích 1 có thể được phủ bởi một hình bình
hành có diện tích 2. 

14


Bài tốn 12. Trong khơng gian cho một số hữu hạn các điểm, mà khơng có
bốn điểm nào trong chúng cùng nằm trên một mặt phẳng sao cho thể tích mỗi
tứ diện tạo ra bởi đỉnh là những điểm này không lớn hơn 1.
Chứng minh rằng tất cả các điểm có thể phủ bởi một tứ diện có thể tích
bằng 27.
Giải.
Giả sử trong tất cả các tứ diện được tạo bởi các điểm đó, tứ diện ABCD là

tứ diện có thể tích lớn nhất.
Qua các đỉnh A, B, C, D dựng các mặt phẳng song song với các mặt đối
diện của đỉnh đó. Lúc đó ta có tứ diện A1B1C1D1.
D1
M

A
B

C
B1

C1
D

A1

Khi đó những điểm A, B, C, D là trọng tâm tương ứng của các tam giác
B1C1D1, AC
1 1D1 , B1 A1D1 , B1C1 A1 .

15


Thật vậy, kéo dài B1C cắt A1D1 tại K, kéo dài B1A cắt C1D1 tại S, kéo dài
B1D cắt C1A1 tại R như hình vẽ dưới đây ta thấy :
CD // KR, CD // C1D1 suy ra KR // SD1
AD // SR, AD // A1D1 suy ra SR // KD1
Do đó SRKD1 là hình bình hành, suy ra SR = KD1.
Chứng minh tương tự ta có SRKA1 là hình bình hành và SR = KA1.

Như vậy K là trung điểm của A1D1.
Tương tự S, R là trung điểm của C1D1 và C1A1.
Do đó có thể kết luận những điểm A, B, C, D là trọng tâm tương ứng của
các tam giác B1C1D1, AC
1 1D1 , B1 A1D1 , B1C1 A1 .
Theo cách này ta nhận được quan hệ tỉ lệ
1 DC
AB
AC
AD
CB
DB
.






3 D 1C1 A 1B1 A 1C1 A 1D1 C 1B1 D 1B1

Từ đây suy ra tỉ số thể tích giữa hai đa diện là

16


Ta sẽ chứng minh tứ diện A1B1C1D1 phủ tất cả các điểm đã cho.
Giả sử tồn tại điểm M trong các
điểm đã cho nằm ngồi tứ diện


D1
M

A1B1C1D1. Khi đó tồn tại ít nhất một
A

đỉnh của đa diện, cho nó là B1 và điểm
M nằm trong hai nửa không gian giác

B

C
B1

C1

nhau như ở hình vẽ trên. Lúc đó

D

VMACD  VABCD .

A1

Điều này trái với việc chọn ABCD là tứ diện có thể tích lớn nhất.
Vậy tứ diện A1B1C1D1 phủ tất cả các điểm của tập hợp đã cho. Và
VA1B1C1D1  27VABCD  27 . Vậy bài toán được chứng minh. 

Bài tốn 13. Chứng minh rằng khơng tồn tại phủ mặt phẳng bằng những
tam giác mà mỗi đỉnh của tam giác là đỉnh của 5 tam giác.

Giải. Giả sử tồn tại một phủ mặt phẳng bằng những tam giác mà mỗi đỉnh
của nó là đỉnh của 5 tam giác. Cho  là một trong những tam giác đó. Những
tam giác có chung đỉnh với  sẽ tạo ra một lục giác B1A1 B2A2 B3A3 như
hình vẽ dưới đây.

17


C3

B3
A2
A3

B2
B1

A1
C2

C1

Như vậy những điểm Ak, k = 1, 2, 3 là những đỉnh của hai tam giác nằm
trong lục giác, còn Bk, k = 1, 2, 3 là những đỉnh của ba tam giác nằm trong lục
giác.
Do đó những điểm Ak, k = 1, 2, 3 còn là những đỉnh của ba tam giác nằm
ngồi lục giác, cịn Bk, k = 1, 2, 3 còn là những đỉnh của hai tam giác nằm
ngoài lục giác.
Như vậy ta nhận được tam giác C1C2C3. Mỗi đỉnh của tam giác này là đỉnh
của 4 tam giác nằm trong C1C2C3. Tổng của bốn góc mà chúng có đỉnh là một

trong các điểm C1, C2, C3 là nhỏ hơn 1800, dễ thấy các điểm đó không thể tiếp
tục là đỉnh của 5 tam giác nữa (xem hình vẽ).
Vậy khơng thể phủ mặt phẳng bằng những tam giác mà mỗi tam giác là
đỉnh của 5 tam giác. 

18


CHƢƠNG 2. BÀI TỐN ĐỒ THỊ, TƠ MÀU
2.1. Lý thuyết cơ bản về bài tốn tơ màu
Khi gặp những bài tốn có nội dung phức tạp, một cơng việc mà ta cần
làm là đưa nó trở về ngơn ngữ tốn học quen thuộc. Điều đó giúp chúng ta dễ
dàng tư duy và việc trình bày lời giải cũng trở nên đơn giản hơn. Một trong
các phương pháp hữu hiệu là phương pháp tô màu. Ta chia các đối tượng
đang xét thành nhiều nhóm đối tượng nhỏ hơn và tơ mỗi nhóm bởi các màu
khác nhau. Kết hợp với các kiến thức hình học và một số phương pháp khác
để giải bài toán.
Một số nguyên lý thường dùng cùng với phương pháp này như nguyên lý
cực hạn, nguyên lý Dirichlet, một số tính chất về các số Ramsey…
Dưới đây tơi trình bày vắn tắt các nội dung này.
Nguyên lý cực hạn. Trong một tập hợp hữu hạn khác rỗng các số thực
luôn tồn tại số nhỏ nhất và số lớn nhất.
Nguyên lý Dirichlet. Nếu nhốt n con thỏ vào k cái lồng, thì tồn tại ít nhất
n
một chuồng có    1 con thỏ.
k 

Định lý 1. Cho k, l là hai số nguyên dương, khi đó tồn tại số nguyên dương
nhỏ nhất R(k;l), sao cho khi ta tô màu tất cả các cạnh của một đồ thị đầy đủ có
R(k;l) đỉnh bởi hai màu xanh và đỏ thì ln tồn tại đồ thị con Kk có tất cả các

cạnh màu đỏ hoặc đồ thị con Kl có tất cả các cạnh màu xanh.
Trên cơ sở định lý này ta chứng minh được một số kết quả quen thuộc.
R(3;3) = 6; R(3;4) = R(4;3) = 9. R(4;4) = 18. R(k,2) = k; R(2;l) = l.
Các kết quả này được áp dụng nhiều trong các bài toán ở phần sau.

19


2.2. Phƣơng pháp tơ màu giải bài tốn hình học
2.2.1. Một số bài tốn tơ màu đồ thị
Ở phần này chúng ta xem xét một số bài tốn hình học sử dụng phương
pháp tơ màu đồ thị. Các bài tốn đưa ra từ mức đơn giản đến phức tạp. Trước
hết ta xem xét một số bài tốn tơ bởi hai màu. Những bài toán này được tham
khảo trong các tài liệu [1], [2], [6] và [7] ở mục tài liệu tham khảo.
Bài toán 14. Cho sáu điểm trong mặt phẳng trong đó khơng có ba điểm nào
thẳng hàng. Các đoạn thẳng nối hai trong sáu điểm đó với nhau được tô bởi màu
xanh hoặc đỏ. Chứng minh luôn tồn tại một tam giác có ba cạnh cùng màu.
Giải.
Gọi sáu điểm đã cho là A, B, C, D, E, F. Điểm A được nối với năm điểm
còn lại bởi năm đoạn thẳng. Năm đoạn thẳng này được tô bởi hai màu. Nên có
ba đoạn thẳng được tơ bởi một màu.
Khơng giảm tính tổng qt ta giả sử ba đoạn thẳng đó là AB, AC, AD và
được tô bởi màu đỏ.
Xét tam giác BCD. Có hai trường hợp xảy ra
B

+ Nếu cả ba cạnh của tam giác BCD
C

được tô bởi màu xanh ta có điều phải

chứng minh.
+ Nếu có một cạnh của tam giác BCD

A
B

được tơ bởi màu đỏ, thì cạnh đó cùng hai

D
C

trong ba cạnh AB, AC, AD tạo thành tam
giác có màu đỏ. 
A

20

D


Bài toán 15. Cho mỗi điểm trên mặt phẳng được tô bằng một trong hai
màu xanh, đỏ. Chứng minh rằng tồn tại một tam giác mà ba đỉnh và trọng tâm
tô cùng màu.
Giải.
Lấy năm điểm tuỳ ý sao cho không có ba điểm nào thẳng hàng trên mặt
phẳng. Khi đó vì chỉ dùng có hai màu để tơ các đỉnh, mà theo nguyên lí
Dirichlet phải tồn tại ba điểm trong số đó cùng màu. Giả sử đó là ba điểm A,
B, C có màu đỏ.
Như vậy ta có tam giác ABC với ba đỉnh màu đỏ.
Gọi G là trọng tâm tam giác ABC. Chỉ có hai khả năng xảy ra:

1. Nếu G có màu đỏ. Khi đó A, B, C, G cùng có màu đỏ và bài tốn đã
được giải.
2. Nếu G có màu xanh. Kéo dài GA, GB, GC các đoạn AA’ = 3GA,
BB’ = 3GB, CC’ = 3GC.
Khi đó, nếu gọi M, N, P tương ứng là các trung điểm của BC, CA, AB thì
A’A = 3AG = 6GM , suy ra A’A = 2AM.
Tương tự B’B = 2BN, CC’ = 2CP.
Do đó các tam giác A’BC, B’AC, C’AB tương ứng nhận A, B, C là trọng
tâm.
Mặt khác, ta cũng có các tam giác ABC và A’B’C’ có cùng trọng tâm G.
Có hai trường hợp sau có thể xảy ra:
a) Nếu A’, B’, C’ cùng có màu xanh. Khi đó tam giác A’B’C’ và trọng tâm
G có cùng màu xanh.

21


b) Nếu ít nhất một trong các điểm A’, B’, C’ có màu đỏ. Khơng mất tính
tổng qt ta giả sử A’ đỏ. Khi đó tam giác A’BC và trọng tâm A màu đỏ.
Vậy trong mọi khả năng luôn tồn tại một tam giác mà ba đỉnh và trọng tâm
cùng màu. 
Bài toán 16. Trên mặt phẳng cho 18 điểm, sao cho khơng có ba điểm nào
thẳng hàng. Nối từng cặp điểm với nhau và tô mỗi cạnh bởi một trong hai
màu xanh hoặc đỏ. Chứng minh rằng ln tìm được một tứ giác mà các đỉnh
của nó nằm trong tập điểm đã cho sao cho cạnh và đường chéo của nó cùng
một màu.
Giải.
Giả sử Ai (i  1,18) là 18 điểm đã cho. Xuất phát từ A1 có 17 đoạn thẳng
A1 Ai (i  2,18) . Mười bảy đoạn thẳng đó chỉ có hai màu xanh hoặc đỏ, nên


theo nguyên lí Dirichlet tồn tại ít nhất 9 đoạn thẳng cùng màu. Khơng giảm
tính tổng qt giả sử đó là các đoạn thẳng A1 A2 , A1 A3 , ..., A1 A10 và chúng cùng
màu đỏ.
Xét 9 điểm A1 , A2 , ..., A10 chỉ có thể xảy ra hai trường hợp sau:
1. Hoặc là tồn tại điểm Aj (2  j  10) sao cho trong 8 đoạn thẳng
Aj Ak (2  k  10, k  j ) có ít nhất bốn đoạn màu đỏ. Khơng mất tính tổng

quát có thể cho là A2 A3 , A2 A4 , A2 A5 , A2 A6 màu đỏ. Đến đây lại chỉ còn hai khả
năng:
+ Hoặc là mọi đoạn thẳng A3 A4 , A3 A5 , A3 A6 , A4 A5 , A4 A6 , A5 A6 đều màu
xanh. Khi đó A3 A4 A5 A6 là tứ giác xanh thoả mãn yêu cầu.
+ Tồn tại một đoạn thẳng Ai , Aj (3  i  j  6) màu đỏ. Khi đó A1 A2 Ai Aj
(3  i  j  6) là tứ giác đỏ thoả mãn yêu cầu bài toán.

22


2. Hoặc là với mọi điểm Aj (2  j  10) , thì trong 8 đoạn thẳng
Aj Ak (2  k  10, k  j) có tối đa ba đoạn màu đỏ mà thơi. Khi đó phải tồn tại

một điểm (chẳng hạn A2 ) mà trong các đoạn A2 Ak (3  k  10, k  j ) có tối
đa hai đoạn màu đỏ thơi. Thật vậy, nếu với mọi Aj (2  j  10) mà có đúng
ba đoạn Aj Ak (2  k  10, k  j ) màu đỏ, thì số đoạn thẳng màu đỏ nối trong
nội bộ 9 điểm đó là

9.3
 13,5 . Vơ lí. Vì A2 Ak (3  k  10, k  j ) có tối đa hai
2

đoạn màu đỏ mà thôi, nên trong số các đoạn A2 A3 , A2 A4 , A2 A5 , A2 A10 có ít

nhất sáu

đoạn màu

xanh. Không

mất

tính

tổng quát ta cho

A2 A5 , A2 A6 , ..., A2 A10 màu xanh.

Xét sáu điểm A5 , A6 , A7 , A8 , A9 , A10 . Đó là sáu điểm mà trong đó khơng có
ba điểm nào thẳng hàng, và mỗi đoạn thẳng nối hai điểm chỉ có hai màu xanh
hoặc đỏ.
Do đó ln ln tồn tại ít nhất một tam giác mà ba đỉnh chọn trong
{ A5 , A6 , A7 , A8 , A9 , A10 } sao cho ba cạnh cùng màu.
Lại có hai khả năng:
a) Giả sử tồn tại tam giác Ai , Aj , Ak (5  i  j  k  10) màu xanh. Khi đó
tứ giác A2 Ai Aj Ak (5  i  j  k  10) là tứ giác xanh thoả mãn yêu cầu đề bài.
b) Nếu tồn tại tam giác Ai , Aj , Ak (5  i  j  k  10) màu đỏ, thì
A1 Ai Aj Ak là tứ giác cần tìm.

Như vậy ta ln chứng minh được tồn tại một tứ giác mà các đỉnh của nó
nằm trong 8 điểm đã cho sao cho cạnh và đường chéo cùng màu. 

23



Bài tốn 17. Cho hình đa giác đều 9 cạnh. Mỗi đỉnh của nó được tơ màu
bằng một trong hai màu trắng hoặc đen. Chứng minh rằng tồn tại hai tam giác
phân biệt có diện tích bằng nhau, mà các đỉnh của mỗi tam giác được tơ cùng
một màu.
Giải.
Gọi chín đỉnh của đa giác là A1, A2 , ..., A9 , và mỗi đỉnh đều được tô bằng
một trong hai màu. Theo ngun lí Dirichlet có ít nhất năm đỉnh trong số đó
được tơ cùng một màu. Giả sử có năm đỉnh được tô màu trắng, năm đỉnh này
tạo ra C53 

5!
 10 tam giác trắng (tam giác trắng là tam giác có ba đỉnh
3!2!

cùng màu trắng).
Gọi  là tập hợp các đỉnh của đa giác đã cho, tức là:
 = { A1, A2 , ..., A9 }

Gọi O là tâm của đa giác đều đã cho (vì là đa giác đều nên luôn tồn tại tâm).
Xét phép quay các góc 00 , 400 , 800 , 1200 , 1600 , 2000 , 2400 , 2800 , 3200
xung quanh tâm O. Rõ ràng ứng với mỗi phép quay này tập  biến thành
chính nó. (Tức là tập các đỉnh của đa giác đều không thay đổi qua phép quay
trên. Mặc dù khi quay thì điểm này biến thành điểm kia.)
Sau 9 phép quay trên thì có 10 tam giác trắng biến thành 90 tam giác trắng.
Mỗi tam giác này đều có các đỉnh thuộc tập hợp  . Chú ý rằng số các tam
giác khác nhau có đỉnh trong  là: C93 

9!
 84 .

6!3!

Vì 84 < 90, nên theo nguyên lí Dirichlet tồn tại hai tam giác trắng  và
 ’ sao cho các phép quay tương ứng cùng một tam giác.

Vì phép quay bảo tồn hình dáng và độ lớn của hình (nói riêng bảo tồn
diện tích), tức là: S  S ' . 

24


Bài tốn 18. (Đề thi vơ địch tốn Trung Quốc 1986). Mỗi điểm trong mặt
phẳng được tô bởi màu đen hoặc màu đỏ. Chứng minh rằng ta có thể tìm được
ba điểm cùng màu mà khoảng cách giữa mỗi cặp điểm bằng 1 hoặc có thể tìm
được ba điểm cùng màu mà khoảng cách giữa các cặp điểm bằng 3 .
D

Giải.
Giả sử không tồn tại ba điểm nào
cùng màu mà mỗi điểm cách nhau bằng

A

B

M

1. Ta sẽ chứng minh ba điểm cùng màu và
mỗi cặp điểm cách nhau 3 .


C

Do ta giả sử không tồn tại ba điểm nào tạo thành tam giác đều có cạnh
bằng 1 được tơ bởi một màu. Nên nếu ta lấy tam giác đều bất kì có cạnh bằng
1 thì có hai đỉnh của tam giác được tô bởi hai màu khác nhau.
Lấy một điểm sao cho khoảng cách tới hai điểm khác màu đó bằng 2. Thì
điểm này phải khác màu với một trong hai điểm nêu trên. Giả sử hai điểm
khác màu có khoảng cách bằng 2 đó là A, B. Gọi M là trung điểm của AB,
MA = MB = 1 và M phải cùng màu với một trong hai điểm A, B.
Giả sử B và M cùng màu đen. Dựng hai tam giác đều MBD và MBC đối
xứng nhau qua AB. Thì các tam giác MBD, MBC đều có cạnh là 1. Do M, B
có cùng màu đen nên D, C phải có màu đỏ. Như vậy ta có tam giác ACD có
các đỉnh được tơ màu đỏ và độ dài cạnh là

3 . Bài toán được chứng minh. 

Bài toán 19. (APMO, 2001) Trong cùng một mặt phẳng cho hai đa giác
đều n cạnh S và T bằng nhau (n ≥ 3). S và T giao nhau tạo thành một đa giác
2n đỉnh. Các cạnh của S được tô màu đỏ, các cạnh của T được tô màu xanh.
Chứng minh rằng tổng độ dài các cạnh màu xanh của đa giác S giao T bằng
tổng độ dài các cạnh màu đỏ của nó.

25


×